0 Daumen
314 Aufrufe

Hallo,
Folgende Aufgabe, es soll die Konvergenz einer Folge gezeigt werden.

an :=\( \frac{4n^3 + (-1)^n + n^2} {5n+2n^3}\)
Das der Grenzwert bei \( \lim\limits_{n\to\infty} \)=2 liegt lässt sich ja durchaus erkennen. Aber wie findent man ein geignetes n0 für die epsilon Umgebung?

Vielen dank im Voraus.

Avatar von

2 Antworten

0 Daumen
 
Beste Antwort

Aloha :)

Forme zunächst den Term für \(a_n\) etwas um, damit eine einfache Abschätzung möglich ist:$$a_n=\frac{4n^3+(-1)^n+n^2}{2n^3+5n}=\frac{4n^3+10n-10n+(-1)^n+n^2}{2n^3+5n}$$$$\phantom{a_n}=2+\frac{n^2-10n+(-1)^n}{2n^3+5n}<2+\frac{n^2}{2n^3+5n}<2+\frac{n^2}{2n^3}=2+\frac{1}{2n}$$Sei nun ein \(\varepsilon>0\) beliebig aber fest gewählt, dann gilt mit \(n_0:=\lceil\frac{2}{\varepsilon}\rceil\)$$\left|a_n-2\right|<\frac{1}{2n}<\varepsilon\quad\text{für alle }n>n_0$$Daher konvergiert die Folge gegen \(2\).

Avatar von 148 k 🚀

Danke das hat mir sehr geholfen!

0 Daumen

Zähler und Nenner durch \( n^3 \) dividieren und \( n \to \infty \) gehen lassen gibt das Ergebnis.

Avatar von 39 k

Ein anderes Problem?

Stell deine Frage

Willkommen bei der Mathelounge! Stell deine Frage einfach und kostenlos

x
Made by a lovely community